LSAT and Law School Admissions Forum

Get expert LSAT preparation and law school admissions advice from PowerScore Test Preparation.

 Administrator
PowerScore Staff
  • PowerScore Staff
  • Posts: 8917
  • Joined: Feb 02, 2011
|
#27401
Complete Question Explanation

Evaluate the Argument. The correct answer choice is (A)

The argument in this stimulus is also causal in nature: the decal program has caused the rate of automobile theft to decrease. As we discussed in question #1, if you want to assess the strength of a causal argument it is important to consider the possibility of alternate causes being present. So the correct answer here will likely provide information suggesting that an alternate cause could potentially be present (in addition to the decals). In other words, once you are told that cars with decals are stolen less often than cars without decals, ask yourself “Why? Was it the decals or something else?” Since that is the questionable element in this argument, the correct answer choice will speak to that uncertainty.

Answer choice (A): This is the correct answer choice. It calls into question the possibility of additional security measures, a clear alternate cause to the decals. You can test the relevance of this answer with the Variance Test: yes, owners with the decals are taking additional security measures (severely weakens the conclusion as the additional measures could have cause the reduced theft rate); no, owners with the decals are not taking additional security measures (strengthens the relationship between decals and lowered theft rates).

Answer choice (B): The number of neighborhoods with the police program is not important, since the argument is only about the neighborhoods where it is already operating (a high number or a low number does not affect the conclusion).

Answer choice (C): Again, the argument is only about the relationship between the decals and the reduced rate of theft. When the cars are stolen does not matter.

Answer choice (D): This is a tempting answer choice for many students, but remember, the conclusion is simply about what has caused the reduction in auto theft in certain cases (decals? something else?). What happens to owners when they drive between 1 AM and 5 AM does not address the relationship in question. That is, if owners are harassed or not, it does not change the potentially validity of the decals deterring thieves from stealing cars.

Answer choice (E): It does not matter if the neighborhoods with the program are representative of all other neighborhoods. The only question is whether the decals caused the observed effect, or if something else might have.
 akanshalsat
  • Posts: 104
  • Joined: Dec 20, 2017
|
#42861
LRTT 8 - EVAL # 1

So I initially thought A, and was about to write that down, but my overthinking got the best of me and I chose C, because I thought that perhaps b/c of the decals, more theft, if occurring in the morning, would make the overall theft from 1-5 go down bc more would be occurring in the morning - which would weaken the argument... so a little confused as to how to say C is wrong. Totally get why A is correct.
 nicholaspavic
PowerScore Staff
  • PowerScore Staff
  • Posts: 271
  • Joined: Jun 12, 2017
|
#43170
Hi akansha,

So with your approach remember that Eval Arg questions help us potentially weaken or potentially strengthen the argument. So with Answer Option (C) when the cars are being stolen, doesn't really matter to the evaluation of the argument because under the Variance Test, we are only concerned about the effectiveness of the program and whether the decals caused the lower rate of theft. I will agree with you that (C) could potentially weaken, but how would knowing that information potentially strengthen the argument? Short answer, it cannot. Thus (A) is the superior choice here.

Thanks for the great question and I hope this helps!

Get the most out of your LSAT Prep Plus subscription.

Analyze and track your performance with our Testing and Analytics Package.